Wie leitet man das Ergebnis des Aharonov-Bohm-Effekts ab?

In den Ableitungen der Aharonov-Bohm- Phase wird dies aufgrund der Einführung des Vektorpotentials direkt erwähnt A , wird eine zusätzliche Phase in die Wellenfunktion für den Fall eingeführt A 0 dh

ψ ( A 0 ) = exp ( ι φ ) ψ ( A = 0 ) ,

Wo

φ = Q P A D X .

Wie man es aus der folgenden Schordinger-Gleichung ableitet

[ 1 2 M ( ich e A ) 2 + v ( R ) ] ψ = ϵ ψ .

Ich habe versucht, die Begriffe zu nehmen, die enthalten A auf der rechten Seite und die Gleichung als inhomogene Gleichung zu behandeln, aber es wird einfach langweilig. Was ist der direkte einfache Weg?

Antworten (2)

Zuerst werde ich einstellen e = 1 der Einfachheit halber.

Lassen ψ 0 bezeichnen die Wellenfunktion, die die freie Schrödinger-Gleichung erfüllt:

(1) ich ψ 0 T = 1 2 M 2 ψ 0 + v ψ 0
Außerdem lassen ψ sei die Wellenfunktion, die der Schrödinger-Gleichung für ein nicht verschwindendes Vektorpotential gehorcht A :
(2) ich ψ T = 1 2 M ( ich A ) 2 ψ + v ψ
Schreiben wir jetzt:
ψ = exp ( ich γ A D l ) ψ 0
Wo γ ist ein Pfad von einem beliebigen Punkt X 0 zu einem anderen Punkt X 1 . Wir können dann schreiben:
( ich A ) 2 ψ = exp ( ich γ A D l ) 2 ψ 0
Setzen Sie diesen Ausdruck in die Gleichung ein ( 2 ) gibt gleichung ( 1 ) . Dies impliziert, dass die Wellenfunktion eines elektrisch geladenen Teilchens, das durch den Raum wandert, wo A 0 erhält eine zusätzliche Phase.

Wir wissen, dass die Wellenfunktion an dem Punkt Q (siehe Abbildung unten) ist ein Ergebnis der Quantensuperposition, dh wir können schreiben:

ψ Q = ψ ( X , γ 1 ) + ψ ( X , γ 2 ) = exp ( ich γ 1 A D l ) ψ 0 ( X , γ 1 ) + exp ( ich γ 2 A D l ) ψ 0 ( X , γ 2 ) = exp ( ich γ 2 A D l ) ( exp ( ich γ 1 A D l ich γ 2 A D l ) ψ 0 ( X , γ 1 ) + ψ 0 ( X , γ 2 ) )
Wir können den Satz von Stoke auf den ersten Term innerhalb der Klammern anwenden, weil γ 1 γ 2 ist ein geschlossener Pfad:
γ 1 A D l γ 2 A D l = B D S = F
Wo F ist der gesamte magnetische Fluss aufgrund des Solenoids durch eine Oberfläche, die durch die geschlossene Begrenzung definiert ist γ 2 γ 1 . Die Wellenfunktion bei Q kann nun geschrieben werden als:
ψ Q = exp ( ich γ 2 A D l ) ( exp ( ich F ) ψ 0 ( X , γ 1 ) + ψ 0 ( X , γ 2 ) )
Dies zeigt, dass die relative Phasendifferenz und damit das Interferenzmuster vom magnetischen Fluss durch die Magnetspule abhängig ist. Dies ist der Aharonov-Bohm-Effekt.

Geben Sie hier die Bildbeschreibung ein

:- die Antwort ist nett. Aber das hast du ja schon vermutet ψ ( A 0 ) hat eine besondere Form. Können wir das besser machen?
@ user38579 Mir ist kein "besseres" Verfahren bekannt.

Um das Problem zu vereinfachen, können wir den Term der potentiellen Energie vernachlässigen v ( R ) , da es für unsere Ableitung einfach irrelevant ist. Also schreiben wir den Hamiltonoperator als

H = 1 2 ( ich X A ) 2 .
Der Grundzustand ist durch Minimierung der Energie gegeben. Da der Hamilton-Operator ein Quadrat von ist ( ich X A ) , also wird es minimiert, wenn ( ich X A ) = 0 . Was bedeutet, dass wir ungefähr den Grundzustand haben
( ich X A ) ψ = 0.
Wenn uns nur die Phasenkonfiguration der Wellenfunktion interessiert, können wir schreiben ψ e ich ϕ , und in die obige Gleichung einsetzen,
( X ϕ A ) e ich ϕ = 0 ,
was bedeutet X ϕ = A , und seine Lösung ist ϕ = A D X .

Hallo Everett. Entschuldigung für die Beule bei dieser alten Antwort. Machen Sie sich keine Sorgen, dass das so definierte \psi nicht einwertig ist, es sei denn, der Fluss ist eine ganze Zahl? Ich fand eine ähnliche Diskussion von Berry in diesem Artikel iopscience.iop.org/article/10.1088/0143-0807/1/3/008
Hallo Ryan, netter Kommentar. Du hast Recht, ψ ist nicht einwertig, was bedeutet, dass die Wellenfront bei Vorhandensein eines nicht ganzzahligen Flusses nicht global definiert ist. Ich denke, eine bessere Möglichkeit, den AB-Effekt zu formulieren, besteht darin, das Pfadintegral zu verwenden, das nicht auf einer global definierten Wellenfront beruht.
Ich bin seit gestern ziemlich weit in diesen Kaninchenbau vorgedrungen! Ich finde es ein faszinierendes Problem. Das ursprüngliche Aharonov-Bohm-Papier spricht sogar darüber, und tatsächlich gaben sie eine einwertige Lösung. Die schönste Ableitung (von Berry), die ich von dieser Lösung gefunden habe, folgt tatsächlich Ihrem Trick der singulären Eichtransformationen, aber in einer Wiederaufnahme der üblichen Winkelerweiterung ist.muni.cz/el/sci/jaro2015/F8592/um/Berry. pdf . Es scheint mit dem integralen Bild des Pfades übereinzustimmen, wie der Pfad den Fluss umkreisen kann, aber ich weiß nicht, wie ich sie direkt in Beziehung setzen soll.
@RyanThorngren Ich verstehe. Danke für den Hinweis auf Berrys nettes Papier. Was ich aus dem Artikel herausgefunden habe, ist, dass die zwei am wenigsten wirbelnden Pfade das Pfadintegral dominieren, da sie die stationärsten Aktionen akkumulieren, sodass es fair ist, die Phasenstruktur durch die singuläre Eichtransformation anzunähern. Aber wenn wir um den Fluss rotieren, werden diese untergeordneten Pfade führend, um das Nicht-Einzelwert-Problem zu lösen. Ich habe das Gefühl, dass der nicht ganzzahlige Fluss zu einer Quantenanomalie bei der SO (2) -Rotation führt, sodass der Drehimpuls Bruchwerte annehmen darf (Symmetriefraktionierung?).
Ja! Wie Sie wissen, gibt es in dem vereinfachten Problem eines Teilchens auf einem Kreis mit einem Fluss darin eine Drehimpulspumpe. Ich denke, dass hier etwas Ähnliches vor sich geht. Es ist eine anomale Sache, weil es am Rande einer 1d Thouless-Pumpe sein möchte. Ich denke, Sie können hier tatsächlich so etwas sehen, wie die Versetzung entlang der Vorwärtsstreurichtung "gepumpt" wird, wenn der Fluss von 0 bis 2pi variiert wird.